2017 AMC 10A Problems/Problem 17

Revision as of 01:06, 24 July 2021 by Mobius247 (talk | contribs) (Solution 2)

Problem

Distinct points $P$, $Q$, $R$, $S$ lie on the circle $x^{2}+y^{2}=25$ and have integer coordinates. The distances $PQ$ and $RS$ are irrational numbers. What is the greatest possible value of the ratio $\frac{PQ}{RS}$?

$\mathrm{\textbf{(A)}}\ 3\qquad\mathrm{\textbf{(B)}}\ 5\qquad\mathrm{\textbf{(C)}}\ 3\sqrt{5}\qquad\mathrm{\textbf{(D)}}\ 7\qquad\mathrm{\textbf{(E)}}\ 5\sqrt{2}$

Solution 1

Because $P$, $Q$, $R$, and $S$ are lattice points, there are only a few coordinates that actually satisfy the equation. The coordinates are $(\pm 3,\pm 4), (\pm 4, \pm 3), (0,\pm 5),$ and $(\pm 5,0).$ We want to maximize $PQ$ and minimize $RS.$ They also have to be non perfect squares, because they are both irrational. The greatest value of $PQ$ happens when $P$ and $Q$ are almost directly across from each other and are in different quadrants. For example, the endpoints of the segment could be $(-4,3)$ and $(3,-4)$ because the two points are almost across from each other. Another possible pair could be $(-4,3)$ and $(5,0)$. To find out which segment is longer, we have to compare the distances from their endpoints to a diameter (which must be the longest possible segment). The closest diameter would be from $(-4,3)$ to $(4,-3)$. The distance between $(3,-4)$ and $(4,-3)$ is shorter than the distance between $(5,0)$ and $(4,-3)$. Therefore, the segment from $(-4,3)$ to $(3,-4)$ is the longest attainable. The least value of $RS$ is when the two endpoints are in the same quadrant and are very close to each other. This can occur when, for example, $R$ is $(3,4)$ and $S$ is $(4,3).$ They are in the same quadrant and no other point on the circle with integer coordinates is closer to the point $(3,4)$ than $(4,3).$ Using the distance formula, we get that $PQ$ is $\sqrt{98}$ and that $RS$ is $\sqrt{2}.$ $\frac{\sqrt{98}}{\sqrt{2}}=\sqrt{49}=\boxed{\mathrm{\textbf{(D)}}\ 7}$

Solution 2

We can look at the option choices. Since we are aiming for the highest possible ratio, let's try using $7$( though actually 5 $\sqrt{2}$ is actually the highest ratio). Now, looking at the problem alone, we know that to have the largest ratio possible, we have to let $RS$ be the minimum possible value while at the same time using integer coordinates. Thus, the smallest possible value of $RS$ is $\sqrt{1^{2}+1^{2}} = \sqrt{2}$. Assuming that $\frac{PQ}{RS} = 7$, we plug in $RS = \sqrt{2}$ and solve for PQ: $PQ=7\sqrt{2}$. Remember, we don't know if this is possible yet, we are only trying to figure out if it is. But for what values of $x$ and $y$ does $\sqrt{x^{2}+y^{2}}=7\sqrt2$? We see that this can easily be made into a $45-45-90$ triangle. But, instead of substituting $y=x$ into the equation and then using a whole lot of algebra, we can save time and use the little trick, that if in a $45-45-90$ triangle, the two $45$ degree sides have side length $s$, then the hypotenuse is $s\sqrt2$. Using this, we can see that $s=7$, and since our equation does in fact yield a sensible solution, we can be assured that our answer is $\boxed{\mathrm{\textbf{(D)}}\ 7}$.

Quality Control by fasterthanlight

(Note by Carrot_Karen: We tried 7, but some might be confused why we concluded that it was the answer after verifying without trying the others, like why wasn't option $\textbf{(E)}$ tried? This is because the problem can only have one correct answer, so if we have an option that already works, we can conclude that none of the others work and $\textbf{(D)}$ is the answer.

Video Solution

https://youtu.be/umr2Aj9ViOA?t=162

See Also

2017 AMC 10A (ProblemsAnswer KeyResources)
Preceded by
Problem 16
Followed by
Problem 18
1 2 3 4 5 6 7 8 9 10 11 12 13 14 15 16 17 18 19 20 21 22 23 24 25
All AMC 10 Problems and Solutions

The problems on this page are copyrighted by the Mathematical Association of America's American Mathematics Competitions. AMC logo.png